X. Électromagnétisme relativiste

Deuxième partie

Transformation des composantes des vecteurs. La charge électrique invariant relativiste. Applications en électromagnétisme relativiste.

1. Transformation des composantes de \(\vec{E},~\vec{H}\)

Rappelons les formules de Lorentz : \[x'=\frac{x-v~t}{\alpha}\quad;\quad y'=y\quad;\quad z'=z\quad;\quad t'=\frac{t-v~x/c^2}{\alpha}\]

Adoptons de nouvelles notations : \[x\rightarrow x^1\quad;\quad y\rightarrow x^2\quad;\quad z\rightarrow x^3\]

Par ailleurs : \[x^0=c~t\quad;\quad\beta=v/c\rightarrow v=\beta~c\]

On conserve les relations : \[x^{2'}=x^2\quad;\quad x^{3'}=x^3\]

En multipliant par \(c\) la quatrième relation de Lorentz en \(t’\), il vient : \[c~t'=\frac{c~t-\beta~c}{\alpha}\qquad\text{soit :}\quad x^{0'}=\frac{x^0-\beta~x^1}{\alpha}\]

La première relation peut s’écrire : \[x'=\frac{x-\beta~c~t}{\alpha}\qquad\text{soit :}\quad x^{1'}=\frac{x^1-\beta~x^0}{\alpha}\]

D’où la matrice de transformation de Lorentz dont les éléments sont notés \((L_{\beta}^{\alpha})'\) : \[\left( \begin{array}{c|cccc} &x^{0'}&x^{1'}&x^{2'}&x^{3'}\\ \hline x^0&1/{\alpha}&-\beta/{\alpha}&0&0\\ x^1&-\beta/{\alpha}&1/{\alpha}&0&0\\ x^2&0&0&1&0\\ x^3&0&0&0&1 \end{array} \right)\]

1.1. Vecteurs \(\vec{E},~\vec{H},~\vec{B},~\vec{D}\)

On cherche les composantes du tenseur \(F^{\alpha\beta}\) dans \(S'^0\) :

\[\begin{aligned} &F^{\alpha'\beta'}=L_{\gamma}^{\alpha'}~L_{\delta}^{\beta'}~F^{\gamma\delta} \\ &E_x=E_1=F_{10}=F^{01} \\ &F^{0'1'}=L_{\gamma}^{0'}~L_{\delta}^{1'}~F^{\gamma\delta} = L_{0}^{0'}~L_{\delta}^{1'}~F^{0\delta}+L_{1}^{0'}~L_{\delta}^{1'}~F^{1\delta} = L_{0}^{0'}~L_{1}^{1'}~F^{01}+L_{1}^{0'}~L_{0}^{1'}~F^{10} \\ &F^{0'1'}=\frac{1}{\alpha^2}~F^{01}+\frac{\beta^2}{\alpha^2}~F^{10} =\frac{1}{\alpha^2}~F^{01}-\frac{\beta^2}{\alpha^2}~F^{01}=\frac{1-\beta^2}{\alpha^2}~F^{01}=F^{01}\end{aligned}\]

On a donc :

\[\begin{aligned} &E'_x=E_x\quad;\quad E_y=F_{20}=F^{02} \\ &F^{0'2'}=L_{\alpha}^{0'}~L_{\beta}^{2'}~F^{\alpha\beta} = L_0^{0'}~L_{\beta}^{2'}~F^{0\beta}+L_1^{0'}~L_{\beta}^{1'}~F^{1\beta} = L_0^{0'}~L_{2}^{2'}~F^{02}+L_1^{0'}~L_{2}^{2'}~F^{12} \\ &F^{0'2'}=\frac{1}{\alpha}~F^{01}-\frac{\beta}{\alpha}~F^{12}\end{aligned}\]

Or, \(F^{12}=c~B_z\). On a donc : \[E'_y=\frac{1}{\alpha}~(E_y-v~B_z)\quad;\quad E'_z=\frac{1}{\alpha}~(E_z-v~B_y)\]

À titre documentaire :

\[\begin{aligned} &B'_x=B_x\quad;\quad B'_y=\frac{1}{\alpha}~(B_y+\frac{v}{c^2}~B_z)\quad;\quad B'_z=\frac{1}{\alpha}~(B_z-\frac{v}{c^2}~E_y) \\ &D'_x=D_x\quad;\quad D'_y=\frac{1}{\alpha}~(D_y-\frac{v}{c^2}~H_z)\quad;\quad D'_z=\frac{1}{\alpha}~(D_z+\frac{v}{c^2}~H_y) \\ &H'_x=H_x\quad;\quad H'_y=\frac{1}{\alpha}~(H_y+v~D_z)\quad;\quad H'_z=\frac{1}{\alpha}~(H_z-v~D_y)\end{aligned}\]

Transformation de courant

\[\begin{aligned} &J^0=\rho~c\quad;\quad J^i=j^i~(j_x,~j_y,~j_z) \\ &J^{\alpha'}=L_{\beta}^{\alpha'}\quad;\quad J^{0'}=A_{\beta}^{0'}~J^{\beta} = L_0^{0'}~J^{0}+L_1^{0'}~J^1 \\ &\rho'~c=\frac{1}{\alpha}~\rho~c-\frac{\beta}{\alpha}j_x\quad\Rightarrow\quad\rho'=\frac{1}{\alpha}~(\rho-\frac{v}{c^2}~j_x) \\ &j^{1'}=L_{\beta}^{1'}~J^{\beta}=L_0^{1'}~J^0+L_1^{1'}~J^1 = -\frac{\beta}{\alpha}~J^0+\frac{1}{\alpha}~J^1\end{aligned}\]

Soit : \[j'_x=\frac{1}{\alpha}~(j_x-v~\rho)\]

On aurait de la même façon : \[j'_x=j_y\quad;\quad j'_z=j_z\]

1.2. La charge électrique est un invariant relativiste.

Considérons un petit volume en un point : \[\Delta q'=\rho'~\Delta v'\]

On suppose la charge fixe dans son repère \(S'\) qui est le repère propre de la charge électrique.

\[x'=\frac{x-v~t}{\alpha}\quad;\quad y'=y\quad;\quad z'=z\]

À l’instant \(t\) de \(S\), on a : \[\Delta x'=\frac{\Delta x}{\alpha}\quad\rightarrow\quad\Delta v'=\frac{\Delta v}{\alpha}\]

Dans \(S’\), il n’y a aucun courant, car la charge y est immobile (\(j'=0\)).

Considérons maintenant la relation précédente : \[\rho'=\frac{1}{\alpha}~(\rho-\frac{v}{c^2}~j_x)\]

Effectuons les transformations : \[v\rightarrow -v\quad;\quad j_x\rightarrow j'_x\quad;\quad\rho'\rightarrow \rho\]

Il vient : \[\rho=\frac{1}{\alpha}~(\rho'+\frac{v}{c^2}~j'_x)=\frac{\rho'}{\alpha}\]

Par suite : \[\Delta q'=\rho'~\Delta v'=\rho~\alpha~\frac{\Delta v}{\alpha}=\Delta q\]

La charge électrique est donc un invariant relativiste. Ainsi, la charge de l’électron est fixe pour tout système. Donc \(e/m\) varie avec la vitesse, car \(m\) varie avec cette vitesse.

2. Quelques applications en électromagnétisme relativiste

2.1. Retour sur la dynamique du point

Rappelons que : \[\overrightarrow{f}=\frac{\overrightarrow{dp}}{dt}\quad;\quad\overrightarrow{p}=\frac{m_0~\overrightarrow{v}}{\alpha}\]

Soit un point \(M\) se déplaçant à une vitesse \(\overrightarrow{v}//\overrightarrow{Ox}\). Nous allons déterminer les expressions des composantes de \(\overrightarrow{f}\) :

\[\begin{aligned} f_x&=\frac{d}{dt}\Big(\frac{m_0}{\alpha}~\frac{dx}{dt}\Big)=\frac{m_0}{\alpha}~\frac{d^2x}{dt^2}- \frac{m_0}{\alpha^2}~\frac{d\alpha}{dt}~\frac{dx}{dt} \\ \alpha^2&=1-\frac{v^2}{c^2}\quad;\quad\alpha~d\alpha=-\frac{v~dv}{c^2} \\ \alpha~\frac{d\alpha}{dt}&=-\frac{v}{c^2}~\frac{dv}{dt}=-\frac{v}{c^2}~\frac{d^2x}{dt^2} \\ f_x&=\frac{m_0}{\alpha}~\frac{d^2x}{dt^2}+\frac{m_0}{\alpha^2}~\frac{1}{\alpha}~\frac{v}{c^2}~\frac{d^2x}{dt^2} \\ f_x&=\frac{m_0}{\alpha}~\frac{d^2x}{dt^2}~\Big(1+\frac{\beta^2}{\alpha^2}\Big)=\frac{m_0}{\alpha^3}~\frac{d^2x}{dt^2}\end{aligned}\]

La quantité \(m_0 / \alpha^3\) est la masse longitudinale de la particule :

\[\begin{aligned} f_l&=\frac{m_0}{\alpha^3}~\gamma_l \\ f_y&=\frac{d}{dt}\Big(\frac{m_0}{\alpha}~\frac{dy}{dt}\Big)=\frac{m_0}{\alpha}~\frac{d^2y}{dt^2}\end{aligned}\]

La quantité \(m_0 / \alpha\) est la masse transversale de la particule : \[f_t=\frac{m_0}{\alpha}~\gamma_t\]

Ainsi, en cinématique relativiste, on établit que : \[\gamma'_l=\frac{\gamma_l}{\alpha^3}\quad;\quad\gamma'_t=\frac{\gamma_t}{\alpha^2}\]

Examinons comment se transforment les forces. Dans \(S'^0\) :

\[\begin{aligned} \overrightarrow{f'}&=m_0~\overrightarrow{\gamma'}\qquad\text{car :}~~v=0 \ \text{à l'instant } t \\ \overrightarrow{f'_t}&=m_0~\overrightarrow{\gamma'_t}\quad;\quad\overrightarrow{f'_l}=m_0~\gamma'_l\end{aligned}\]

On a donc : \[f'_t=\frac{f_t}{\alpha}\quad;\quad f'_l=f_l\]

2.2. Découverte de la formule de Laplace

On se place dans le repère propre de la particule \(S’\) (\(M\) y est au repos) ; on sait que la charge est invariante \(q’=q\). On suppose qu’il existe un champ \(\overrightarrow{E'}\) dans \(S’\), donc une force agit sur cette particule : \[\overrightarrow{F'}=q~\overrightarrow{E'}\quad\rightarrow\quad\overrightarrow{F'_l}=q~\overrightarrow{E'_l}~~~;~~~ \overrightarrow{F'_t}=q~\overrightarrow{E'_t}\]

Examinons ce que deviennent ces formules dans \(S\) : \[E'_x=E_x\quad\rightarrow\quad\overrightarrow{E'_l}=\overrightarrow{E_l}\]

\[\begin{aligned} E'_y=\frac{1}{\alpha}~(E_y-v~B_z)\\ E'_z=\frac{1}{\alpha}~(E_z+v~B_y) \end{aligned}\quad\rightarrow\quad E'_t=E'_x+E'_y=\frac{1}{\alpha}~(\overrightarrow{E_t}+\overrightarrow{v}\wedge\overrightarrow{B})\]

\[\overrightarrow{F_l}=q~\overrightarrow{E_l}\quad;\quad\frac{\overrightarrow{F_t}}{\alpha}=\frac{q}{\alpha} ~(\overrightarrow{E_t}+\overrightarrow{v}\wedge\beta)\]

En additionnant, on retrouve la formule bien connue : \[\overrightarrow{F}=\overrightarrow{F_l}+\overrightarrow{F_t}=q~(\overrightarrow{E}+\overrightarrow{v}\wedge\overrightarrow{B})\]

Il y a donc bien unité entre l’électrostatique et l’électromagnétisme.

2.3. Découverte de la loi de Biot et Savart

On considère un point \(P\) de \(S'\) et on pose \(MP=r'\) ; \(M\) crée un champ \(\overrightarrow{E'}\) en \(P\) : \[\overrightarrow{E'}=\frac{q}{4\pi~\varepsilon_0}~\frac{\overrightarrow{r'}}{r^3}\quad;\quad\overrightarrow{B'}=\overrightarrow{0}\]

On sait que :

\[\begin{aligned} &B_x=B'_x \\ &B_y=\frac{1}{\alpha}~(B'_y-\frac{v}{c^2}~E'_z) \\ &B_y=\frac{1}{\alpha}~(B'_z+\frac{v}{c^2}~E'_y)\end{aligned}\]

On a donc : \[\overrightarrow{B_l}=\overrightarrow{B'_l}\quad;\quad\overrightarrow{B_t}=\frac{1}{\alpha}~(\overrightarrow{B'_t}+\frac{\overrightarrow{v}}{c^2}\wedge\overrightarrow{E'})\]

Or ici \(\overrightarrow{B'}=\overrightarrow{0}\). On a donc : \[\overrightarrow{B}=\overrightarrow{B_t}=\frac{1}{\alpha}~(\frac{\overrightarrow{v}}{c^2}\wedge\overrightarrow{E'})\] \[\overrightarrow{B}=\frac{1}{\alpha}~\frac{q}{4\pi~\varepsilon_0}~\frac{\overrightarrow{v}}{c^2}\wedge\frac{\overrightarrow{r'}}{r'^3}\]

Comme \(\mu_0~\varepsilon_0~c^2=1\), il vient : \[\overrightarrow{B}=\frac{1}{\alpha}~\frac{\mu_0}{4\pi}~q~\overrightarrow{v}\wedge\frac{\overrightarrow{r'}}{r'^3}\]

On a donc trouvé l’existence d’une induction \(B\). Cette formule ressemble tout à fait à celle de la loi de Biot et Savart, formule que l’on retrouve en postulant que \(\alpha=1\) et \(r=r'\) : \[\overrightarrow{B}=\frac{\mu_0}{4\pi}~i~\overrightarrow{dl}\wedge\frac{\overrightarrow{r}}{r^3}\]

Code QR de la page

Télécharger

↑ Haut